Calculate the arc length of the vector function

Click For Summary
To calculate the arc length of the vector function <2t, t^2, lnt> from t=1 to t=e, the arc length formula is applied, resulting in the integral ∫√{4 + 4t^2 + 1/t^2} dt. The integrand needs to be rewritten for clarity, suggesting the expression can be factored as (2t + 1/t)^2. This simplification aids in the integration process. The discussion emphasizes the importance of including dt in the integral and offers tips for factoring the expression correctly. Ultimately, the participants confirm the correct approach to solving the arc length problem.
jaydnul
Messages
558
Reaction score
15

Homework Statement


Calculate the arc length of &lt;2t,t^2,lnt&gt; from 1=&lt;t=&lt;e

Homework Equations


Arc length=∫√{(x&#039;)^2 + (y&#039;)^2 + (z&#039;)^2}

The Attempt at a Solution


So I have gotten to this point:
∫√{4 + 4t^2 + \frac{1}{t^2}}

Am I on the right track, and if so, how do I integrate that?
 
Physics news on Phys.org
Jd0g33 said:

Homework Statement


Calculate the arc length of &lt;2t,t^2,lnt&gt; from 1=&lt;t=&lt;e

Homework Equations


Arc length=∫√{(x&#039;)^2 + (y&#039;)^2 + (z&#039;)^2}
Missing dt in the integrand.

Also, if you use \sqrt instead of √, it will look better.
$$\int \sqrt{x'^2 + y'^2 + z'^2}dt $$
Jd0g33 said:

The Attempt at a Solution


So I have gotten to this point:
∫√{4 + 4t^2 + \frac{1}{t^2}}

Am I on the right track, and if so, how do I integrate that?
Write the quantity in the radical as 4t2 + 4 + 1/t2, and then factor it.
 
Thanks. Good advice so far. I think I might just be an idiot by how do you factor that? Haven't had to do that for a while.
 
Nevermind, got it. multiply t^2
 
Thanks for the help!
 
Jd0g33 said:
Nevermind, got it. multiply t^2
You can't do that, but you can multiply by t2 over itself. OTOH, the expression can be factored directly, to (2t + 1/t)2.
 
Question: A clock's minute hand has length 4 and its hour hand has length 3. What is the distance between the tips at the moment when it is increasing most rapidly?(Putnam Exam Question) Answer: Making assumption that both the hands moves at constant angular velocities, the answer is ## \sqrt{7} .## But don't you think this assumption is somewhat doubtful and wrong?

Similar threads

  • · Replies 2 ·
Replies
2
Views
2K
Replies
2
Views
2K
Replies
5
Views
2K
  • · Replies 5 ·
Replies
5
Views
2K
  • · Replies 5 ·
Replies
5
Views
2K
  • · Replies 1 ·
Replies
1
Views
2K
  • · Replies 6 ·
Replies
6
Views
2K
Replies
3
Views
2K
Replies
1
Views
1K
  • · Replies 6 ·
Replies
6
Views
2K